for f(x)=3x, find f(4) and f(-3)​

Answers

Answer 1
For f(4)=3x the answer is 12 and for f(-3)=3x the answer is -9

Related Questions

If a statistic used to estimate a parameter is said to be unbiased, then which of the following must be true? (A) The statistic is equal to the true value of the parameter it is being used to estimate for every possible sample. (B)The mean of the sampling distribution of the statistic is equal to the true value of the parameter it is being used to estimate. (C) The sampling distribution of the statistic has the same mean and standard deviation as the distribution of the population. (D)The statistic is a proportion. (E) The mean of the sampling distribution of the statistic will change as the sample size is changed.

Answers

If a statistic used to estimate a parameter is said to be unbiased, then the mean of the sampling distribution of the statistic is equal to the true value of the parameter it is being used to estimate (B).

In statistics, when a sample statistic is unbiased, it means that the statistic does not deviate from the population parameter that it is trying to estimate. The unbiased estimator is not consistently overestimating or underestimate the true population parameter value.

It is measured by the mean of the sampling distribution. In other words, if we take every possible sample of the same size n from the same population and calculate the statistic, the mean of those statistics would be equal to the true value of the parameter being estimated.

Option (A) is not true, because it is not necessary for the statistic to be exactly equal to the true value of the parameter for every possible sample, only that the average of the statistic over many samples is equal to the true value.

Option (C) is not necessarily true, as the sampling distribution of the statistic may have a different standard deviation than the population distribution.

Option (D) is not true, as unbiasedness applies to any type of statistic, not just proportions.

Option (E) is not true, as the mean of the sampling distribution of an unbiased statistic does not depend on the sample size, only on the true value of the parameter being estimated.

Therefore, the correct option is (B) The mean of the sampling distribution of the statistic is equal to the true value of the parameter it is being used to estimate.

To know more about the "mean of the sampling distribution": https://brainly.com/question/26952915

#SPJ11

the tree nearest the house is our starting point. our point person is taking the clinometer reading 15.24 meters from the tree's base and they get a reading of 237`. our point person is 1.83 meters in height. how tall is the tree, rounded to the nearest meter?

Answers

he tree nearest the house is our starting point. Our point person is taking the clinometer reading 15.24 meters from the tree's base and they get a reading of 237`. Our point person is 1.83 meters in height.

How tall is the tree, rounded to the nearest meter?The height of the tree can be determined by using the tangent formula. The tangent formula is tan θ = h/d where θ = angle of elevation, h = height of the object, and d = horizontal distance.

The clinometer reading is the angle of elevation. Hence, we can use the given data to determine the height of the tree.The point person is standing at 15.24 m from the base of the tree. Therefore, the horizontal distance (d) is 15.24 m. The angle of elevation (θ) is 237 degrees (given in the question).

Convert the degrees to radians as tan function uses radians. Convert degrees to radians:[tex]237 × (π/180) = 4.135[/tex]radians.Now we can use the tangent formula to determine the height of the tree:tan θ = h/dtan 4.135 = [tex]h/15.24h = 15.24 × tan 4.135h = 15.24 × 0.07311h ≈ 1.1132[/tex] metersThe height of the tree is 1.1132 meters. But, we have to round the answer to the nearest meter. Therefore, the height of the tree, rounded to the nearest meter, is 1 meter.

for such more questions on The angle of elevation

https://brainly.com/question/88158

#SPJ11

For some real number $a$ and some positive integer $n$, the first few terms in the expansion of $(1 + ax)^n$ are

Answers

Given expression is $(1 + ax)^n$If $n$ is a positive integer then by using binomial theorem,

we can expand the given expression as$(1 + ax)^n$ $= 1 + nax + \frac{n(n - 1)}{2!}(ax)^2 + \frac{n(n - 1)(n - 2)}{3!}(ax)^3 + ... $Let the first few terms in the expansion of $(1 + ax)^n$ be$1 + 4ax + 6a^2x^2 + 4a^3x^3 + a^4x^4$Thus,$1 = \binom{n}{0}$ $⇒ nC_0 = 1$$nax = \binom{n}{1}$ $⇒ nC_1a^1x^1 = 4a$x $⇒ n = 4$ $\ \ \ \ $ $⇒ a = \frac{1}{4x}$The given expression becomes $\left(1 + \frac{x}{4}\right)^4$

Expanding this using binomial theorem, we get$\left(1 + \frac{x}{4}\right)^4$ $= \binom{4}{0}1^4\left(\frac{x}{4}\right)^0 + \binom{4}{1}1^3\left(\frac{x}{4}\right)^1 + \binom{4}{2}1^2\left(\frac{x}{4}\right)^2 + \binom{4}{3}1^1\left(\frac{x}{4}\right)^3 + \binom{4}{4}1^0\left(\frac{x}{4}\right)^4$$= 1 + x + 3x^2 + 4x^3 + \frac{1}{4}x^4$

Hence, the first few terms in the expansion of $(1 + ax)^n$ are $1 + 4ax + 6a^2x^2 + 4a^3x^3 + a^4x^4$ where $n = 4$ and $a = \frac{1}{4x}$. The expression can be further simplified to $1 + x + 3x^2 + 4x^3 + \frac{1}{4}x^4$.

for such more questions on binomial theorem

https://brainly.com/question/15368291

#SPJ11

Math 4th 11-4 I need answers for 11-4 can you please help?

Answers

To make the table of 7, using the table of 4 and 3, we add the value of both table consecutively.

We have to make the table of 7, using table of 4 and 3.

As we know the table of 3 is:

3  6  9  12  15  18  21  24  27  30

As we know the table of 4 is:

4  8  12  16  20  24  28  32  36  40

To from the table of 7 using the table of 4 and 3 we add the consecutive value of both table respectively.

3 + 4    6 + 8    9 + 12    12 + 16    15 + 20    18 + 24    21 + 28    24 + 32      27 + 36    30+40

Now simplify

7    14    21    28    35    42    49    56    63    70

To learn more about addition link is here

brainly.com/question/29560851

#SPJ4

The complete question is:

Math 4th 11-4: Help bunty to make the table of 7, using table of 4 and 3.

leila is on her way home in her car. She has driven 30 miles so far, which is one-third of the way home. what is the total length of her drive.​

Answers

Answer:

90 miles

Step-by-step explanation:

We know

She has driven 30 miles so far, which is one-third of the way home.

What is the total length of her drive?

We Take

30 x 3 = 90 miles

So, the total length of her drive is 90 miles.

What is the sum of the two amounts of money shown?

Answers

Answer:

$21.39

Step-by-step explanation:

The way we notate money is by putting dollars first, and then cents at the end of the decimal.

For the first amount of money, we have 15 dollars in cash. We have 3 quarters, and each quarter is 25 cents. If we add 25 cents 3 times, we get 75 cents. We are then given 2 nickels, which are 5 cents. We add 10 to 75, and we get 85 cents.

[tex]15.85[/tex]

We are then give a singular 5 dollar bill. We also have two quarters, which are again 50 cents. Add 4 to this, and we get 54.

[tex]5.54[/tex]

Now we add the two together!

[tex]15.85+5.54=21.39\\[/tex]

the time it takes for a statistics professor to grade an exam is normally distributed with a mean of 9.7 minutes and a standard deviation of 1.9 minutes. there are 50 students in the professor's class. what is the probability that more than 8 hours are needed to grade all of the exams? (report your answer to 4 decimal places.)

Answers

The probability that more than 8 hours are needed to grade all of the exams is about 52%

What is the probability of a standard normal distribution?

The probability of a standard normal distribution is the area under the curve of the normal distribution function within a specified interval.

Let X represent the random variable to grade an exam, and let Y represent the total time to grade all exams

The number of students = 50

Therefore;

Y = 50·X

The properties of the normal distribution indicates that we get;

E(Y) = E(50·X) = 50·E(X) = 50 × 9.7 = 485

Var(Y) = Var(50·X) = 50²·Var(X) = 50² × 1.92² = 9025

The standard deviation, SD(Y) = √(Var(Y)) = √(9025) = 95

The probability that more than 8 hours are needed can be found using the z-score of the normal distribution as follows;

8 hours = 480 minutes

Z = (480 - 485)/95 ≈ -0.0526

The probability obtained from a standard normal table, is therefore;

P(Z > -0.0526) = 1 - 0.48006 ≈ 0.52

The probability that more than 8 hours are needed to grade all students is therefore about 0.52 or 52%

Learn more about the standard normal table here: https://brainly.com/question/28991504

#SPJ1

Write as an expression the difference of 7 and twice the product of a and b

Answers

The expression represents the difference of 7 and twice the product of a and b is 7 - 2ab

Let's break down the given problem step by step. First, we need to find the product of a and b, which is done by multiplying the two variables together using the multiplication symbol (*). Then, we need to multiply this result by 2, which is done by placing the entire product inside parentheses and then multiplying it by 2 using the multiplication symbol again.

Once we have found twice the product of a and b, we need to subtract it from 7. This can be done using the subtraction symbol (-), which we place between 7 and the expression we just found.

Putting it all together, the expression we get is:

7 - 2ab

where a and b are the two variables we were given.

To know more about expression here

https://brainly.com/question/14083225

#SPJ4

The obtuse angle between the hands of a clock at 2.30 a.m. is

Answers

Answer: 105

Step-by-step explanation:

OBTUSE ANGLE (105°) IS FORMED AT 2:30 IN THE CLØCK.

As we know 12 division OF 360° CIRCLE GIVES 30°

AT TWO PM THE ANGLE BETWEEN HOUR AND MINUTE HAND IS 60° AFTER 30 MIN ARM ROTATES 180° AND HOUR ARM 30°/2 = 15°

SO THE ANGLE BETWEEN TWO ARMS IS (180-(60-15)

105

Below is a list of all possible outcomes in the experiment of rolling two die. 1.2 1,3 14 15 1,6 21 22 23 24 25 2,6 34B2 33 3,4 3 5 3.6 41 4 2 43 4,4 4 5 4,6 5 52 33 5 4 5,5 56 6,1 6,2 6.3 6 4 6,5 6.6 Determine the following probabilities. Write your answers as reduced fractions_ P(sum is odd) P(sum is 5) P(sum is 7) = P(sum is 7 and at least one of the die is a 1) = 18 P(sum is 7 or at least one of the die is 1) = 36

Answers

Thus, the following outcomes satisfy the condition:1, 61, 11, 12, 21, 13, 31, 14, 41, 15, 52, 25, 34, and 43Therefore, the probability of the sum being 7 or at least one die being 1 is:P(sum is 7 or at least one die is 1) = 15/36 = 5/12

Hence, P(sum is odd) = 7/36, P(sum is 5) = 1/9, P(sum is 7) = 1/6, P(sum is 7 and at least one die is 1) = 5/18, and P(sum is 7 or at least one die is 1) = 5/12.

In the given experiment of rolling two dice, the following probabilities are to be determined:

P(sum is odd), P(sum is 5), P(sum is 7), P(sum is 7 and at least one of the die is 1), and P(sum is 7 or at least one of the die is 1).The sum of two dice is odd if one die has an odd number and the other has an even number. The possibilities of odd numbers are 1, 3, and 5, while the possibilities of even numbers are 2, 4, and 6. Therefore, the following outcomes satisfy the condition:

1, 22, 24, 36, 42, 44, and 66Thus, the probability of the sum being odd is: P(sum is odd) = 7/36The sum of two dice is 5 if one die has 1 and the other has 4, or one die has 2 and the other has 3. Thus, the following outcomes satisfy the condition:1, 42, 3Therefore, the probability of the sum being 5 is: P(sum is 5) = 4/36 = 1/9The sum of two dice is 7 if the dice show 1 and 6, 2 and 5, 3 and 4, 4 and 3, 5 and 2, or 6 and 1.

Thus, the following outcomes satisfy the condition:1, 63, 54, 45, 36, and 2Therefore, the probability of the sum being 7 is: P(sum is 7) = 6/36 = 1/6The sum of two dice is 7 and at least one die is 1 if the dice show 1 and 6, 6 and 1, 1 and 1, 1 and 2, 2 and 1, 1 and 3, 3 and 1, 1 and 4, 4 and 1, or 1 and 5. Thus, the following outcomes satisfy the condition:1, 61, 11, 12, 21, 13, 31, 14, 41, and 15

Therefore, the probability of the sum being 7 and at least one die being 1 is:P(sum is 7 and at least one die is 1) = 10/36 = 5/18The sum of two dice is 7 or at least one die is 1 if the dice show 1 and 6, 6 and 1, 1 and 1, 1 and 2, 2 and 1, 1 and 3, 3 and 1, 1 and 4, 4 and 1, 1 and 5, 2 and 5, 5 and 2, 3 and 4, or 4 and 3.

for such more questions on probabilities

https://brainly.com/question/13604758

#SPJ11

Leo has a number of toy soldiers between 27 and 54. If he wants to group them four by four, there are none left, seven by seven, 6 remain, five by five, 3 remain. How many toy soldiers are there?

The answer is 48 but I need step by step explanation

please help it’s due today(midnight right now), I will mark brainliest

Answers

There are 48 toy soldiers, which is 6 x 8 of them.

How did Like Toy Soldiers come to be?

The anger Eminem expresses in "Like Toy Soldiers" is a result of his personal beefs with rappers Ja Rule and Benzino, who was the editor of The Source at the time. The song, "Toy Soldiers," by Martika, was sampled on the 2004 release Encore.

Let's name Leo's collection of toy soldiers "x" the amount.

As a result of the problem statement, we are aware of:

There are no more when he arranges them in groups of four, proving that x is divisible by four.

Six remain after he divides them into groups of seven, proving that (x - 6) is divisible by seven.

He organizes them into fives. If there are still 3 after multiplying by 5, (x - 3) can be divided by 5.

We may create a system of equations based on these three conditions:

x = 4a (from the first condition)

x - 6 = 7b (from the second condition)

x - 3 = 5c (from the third condition)

where a, b, and c are integers.

4a - 6 = 7b

4a - 3 = 5c

Now we need to solve for a, b, and c.

7b = 4a - 6

7b + 6 = 4a

Since 7 and 4 are relatively prime, we know that (7b + 6) must be divisible by 4. Therefore, we can write:

7b + 6 = 4k

where k is some integer. Solving for b, we get:

b = (4k - 6) / 7

Since b is an integer, k must be 2, which gives us:

b = (4(2) - 6) / 7 = -1

We can try the next possible value of k, which is 3:

b = (4(3) - 6) / 7 = 0

x - 3 = 5c

6 - 3 = 5c

c = 1

6 divided by 4 is 1 with no remainder.

(6 - 6) divided by 7 is 0 with a remainder of 0.

(6 - 3) divided by 5 is 1 with a remainder of 0.

Therefore, the answer is 48, which is 6 times 8.

To know more about integer visit:-

https://brainly.com/question/15276410

#SPJ1

Can anyone please solve this math problem? Thanks!

Answers

Therefore , the solution of the given problem of surface area comes out to be 9664 mm².

What precisely is a surface area?

Its total size can be determined by figuring out how much room would be required to completely cover the outside. When choosing comparable substance with a rectangular shape, the surroundings are taken into account. Something's total dimensions are determined by its surface area. The volume of water that a cuboid can contain depends on the number of edges that are present in the region between its four trapezoidal angles.

Here,

We are aware that this total area is equivalent to the white cross's area times four.

Therefore:

Total flag area minus the crimson background area equals the area of the white cross.

=> 13,348 - 4x equals area of white cross

When we use this expression in the preceding equation as the area of the white cross, we obtain:

=>  4x + (13,348 - 4x) = 13,348

When we simplify and find x, we obtain:

=>  x = 836

As a result, the size of the white cross is: 836 mm2, the size of each parallelogram containing the fleur-de-lis is:

=> Area of white cross = Total area of flag - Area of red background

=>  13,348 - 4x

=>  13,348 - 4(836)

=>  9664 mm²

To know more about surface area visit:

brainly.com/question/2835293

#SPJ1

A circular flower garden has an area of 314m². A sprinkler at the center of the garden can cover an area of 12 m. Will the sprinkler water the entire garden?

Answers

Step-by-step explanation:

No,

if the sprinkler covers a distance of 12 m meaning the 12 m is the diameter...then to find the area that it covers we use the formula for the circle since it's circular

A=πr2

A=3.142*36

A=113.112 cm3

An angle measures 174.8° more than the measure of its supplementary angle. What is the measure of each angle?


I need it before 3 / 11 / 23 please

Answers

Answer:

Step-by-step explanation:

Let x be the measure of the smaller angle.

Since the two angles are supplementary, we have:

x + (x + 174.8°) = 180°

Simplifying the equation gives:

2x + 174.8° = 180°

Subtracting 174.8° from both sides gives:

2x = 5.2°

Dividing by 2 gives:

x = 2.6°

Therefore, the smaller angle measures 2.6°, and the larger angle (which is supplementary) measures:

x + 174.8° = 2.6° + 174.8° = 177.4°

PLEASE HELP FIRST CORRECT WILL GET BRAINLIEST

Answers

Answer: Felipe has walked 25.1 meters.

Step-by-step explanation:

Felipe walks the length of his living room, which is 9.1 meters. He then turns and walks the width of his living room, which is 3.5 meters. Finally, he walks back to the corner he started from, which is another 9.1 meters.

The total distance that Felipe has walked is the sum of the distances he covered in each of these three parts of his walk. So, we need to add up 9.1 meters, 3.5 meters, and 9.1 meters to get the total distance.

9.1 m + 3.5 m + 9.1 m = 21.7 m

Therefore, Felipe has walked 21.7 meters so far. However, he still needs to walk back to the corner he started from. This distance is equal to the diagonal of the rectangle formed by his living room.

We can use the Pythagorean theorem to find the length of this diagonal. The length and width of the rectangle are 9.1 meters and 3.5 meters, respectively. Let d be the length of the diagonal, then:

d² = 9.1² + 3.5²

d² = 83.06

d ≈ 9.11 meters

Therefore, the total distance that Felipe has walked is approximately:

21.7 m + 9.11 m ≈ 25.1 m

So, Felipe has walked about 25.1 meters.

Answer:

Felipe has walked 25.2 meters in total.

Step-by-step explanation:

To find out how far Felipe has walked, we need to calculate the perimeter of his living room. The perimeter is the distance around the outside of a shape.

The formula for the perimeter of a rectangle is:

perimeter = 2(length + width)

Given that the length of Felipe's living room is 9.1 meters and the width is 3.5 meters, we can substitute these values into the formula and get:

perimeter = 2(9.1 + 3.5)

perimeter = 2(12.6)

perimeter = 25.2 meters

Please I will give brainliest
Using one of the endpoints of the diameter and center of the circle write an equation of the circle ​

Answers

The equation of the circle graphed in this problem is given as follows:

x² + y² = 34.

What is the equation of a circle?

The equation of a circle of center [tex](x_0, y_0)[/tex] and radius r is given by:

[tex](x - x_0)^2 + (y - y_0)^2 = r^2[/tex]

The radius of a circle represents the distance between the center of the circle and a point on the circumference of the circle, while the diameter of the circle is the distance between two points on the circumference of the circle that pass through the center. Hence, the diameter’s length is twice the radius length.

The center of the circle is at the origin, hence:

x² + y² = r².

The diameter of the circle is given by segment AC, which is the hypotenuse of a right triangle of sides 10 and 6, hence it's length is obtained applying the Pythagorean Theroem as follows:

d² = 10² + 6²

d = sqrt(10² + 6²)

d = 11.66.

The radius is half the diameter, hence:

r = 11.66/2

r = 5.83.

Then the equation of the circle is given as follows:

x² + y² = 5.83²

x² + y² = 34.

More can be learned about the equation of a circle at https://brainly.com/question/1506955

#SPJ1

The number of paintings owned by an art museum was 200. Since this time, the collection of paintings has grown by 2% each month. Which expressions represent the number c
paintings owned by the art museum 2 years later if it continues to grow at this rate?
A 200-(1+0.02¹2) 24
B 200-(1.02)²
C 200- 1+0.02)¹²) ²
D 200• (1.02)^24
E 200• (0.02) ^24

Answers

Therefore , the solution of the given problem of expressions comes out to be choice D is the correct response: 200 • (1.02)24.

What is an expression ?

It is preferable to use moving numbers, which can be growing decreasing, or variable, rather than generating estimates at random. They could only assist one another by exchanging resources, knowledge, or answers to problems. A truth statement may contain strategies, components, and notations against mathematical processes such as additional denial, synthesis, and mixture.

Here,

The number of paintings in the library is increasing by 2% each month, so the growth rate for one month is 2/100 = 0.02.

Therefore, multiplying the starting number of paintings (200) by the growth factor

=>  (1 + 0.02)24,

where 24 is the number of months in 2 years, will give the number of paintings the art museum will own after two years (24 months).

Thus, the expression that denotes the number of paintings the art institution owns after two years is as follows:

=>  D) 200 • (1.02)^24

Therefore, choice D is the correct response: 200 • (1.02)24.

To know more about expressions visit :-

brainly.com/question/14083225

#SPJ1

A production facility contains two machines that are used to rework items that are initially defective. Let X be the number of hours that the first machine is in use, and let Ybe the number of hours that the second machine is in use, on a randomly chosen day. Assume that X and Y have joint probability density function given by 3. f(x) = { 3/2(x^2 +y^2) 0

Answers

Answer:

Step-by-step explanation:

PLSSSS HELP IF YOU TURLY KNOW THISSS

Answers

Answer: x = 18

Step-by-step explanation:

x - 3 = 15

x = 15 + 3

x = 18

[tex]\huge{\color{pink}{\underline{\color{pink}{\underline{\color{cyan}{\textbf{\textsf{\colorbox{purple}{Answer ≈}}}}}}}}}[/tex]

x = 18

Step-by-step explanation:

Given ,

≈> x – 3 = 15

•add 3 on both the sides

≈> x –3 + 3 = 15 + 3

≈> x = 18

Hope it helps you :)

PLEASE !! HELP i’ve been on this question for a while

Answers

Answer:

y = - [x - 1] + 2

Step-by-step explanation:

I think this is right

Im sorry if not :)

A box contains some green and yellow counters. 7/9of the box is green counters. Are 24 yellow counters. There How many green counters are there?

Answers

If 7/9 of the box is green counters, and there are 24 yellow counters in the box, then there are 84 green counters .

Let's assume that the total number of counters in the box is x.

We are given that 7/9 of the box is filled with green counters, which means that the remaining 2/9 of the box must be filled with yellow counters. We are also given that there are 24 yellow counters in the box.

We can set up an equation to represent the relationship between the number of yellow counters and the total number of counters:

2/9 x = 24

To solve for x, we can multiply both sides of the equation by the reciprocal of 2/9, which is 9/2:

(2/9) x * (9/2) = 24 * (9/2)

x = 108

This means that there are a total of 108 counters in the box. To find out how many of these are green counters, we can use the fact that 7/9 of the box is filled with green counters:

(7/9) * 108 = 84

To learn more about counters click on,

https://brainly.com/question/30609303

#SPJ4

Find an equation of the line drawn below.

Answers

The equation of the line is x=-2

Answer:

x = - 2

Step-by-step explanation:

the equation of a vertical line parallel to the y- axis is

x = c ( c is the value of the x- coordinates the line passes through )

the line passes through all points with an x- coordinate of - 2 , then

x = - 2 ← equation of line

Use the following circle to find the indicated measure.

MK
is a diameter.

Find m ∠
LKM

Answers

The answer of the given question based on finding the m∠LKM from the given circle the answer is the measure of ∠LKM is 140° degrees.

What is Diameter?

In geometry,  diameter of circle is  line segment that passes through  center of  circle and has both endpoints on circle. The diameter is the longest chord (line segment connecting two points on  circumference) of  circle. The length of  diameter is twice the length of radius, which is distance from the center of  circle to any point on  circumference.

The diameter i important property of a circle and is used to calculate other properties, like the circumference and area of the circle

Since MK is a diameter of the circle, it passes through the center of the circle, which we can label as point O. Therefore, ∠LKM is an inscribed angle that intercepts arc LM.

By the Inscribed Angle Theorem, we know that the measure of an inscribed angle is equal to half the measure of the arc that it intercepts. Therefore, to find the measure of ∠LKM, we need to find the measure of arc LM.

We are given that the measure of arc LK is 100° degrees. Since arc LM is the sum of arcs LK and KM, and MK is a diameter (so arc KM is also a semicircle with a measure of 180 degrees), we can write:

m(arc LM) = m(arc LK) + m(arc KM) = 100 + 180 = 280° degrees

Therefore, the measure of ∠LKM is:

m∠LKM = 1/2 * m(arc LM) = 1/2 * 280 = 140° degrees

So the measure of ∠LKM is 140° degrees.

To know more about  Inscribed Angle Theorem visit:

https://brainly.com/question/5436956

#SPJ1

Jim orders prints from a website. The site charges him $6. 95 a month and $0. 04 for each print he orders.

Enter an equation that can be used to find the number of prints, P, Jim ordered last month if the website charged

him $17. 79. Enter your response in the first response box

Enter the number of prints Jim ordered last month. Enter your response in the second response box,

Answers

Number of  prints Jim ordered last month is 271

Let's assume Jim ordered "P" prints last month. The cost of ordering "P" prints would be the sum of the monthly charge and the cost of each print, which is given by the equation:

Cost = Monthly Charge + (Cost per Print x Number of Prints)

Substitute the values in the equation

$17.79 = $6.95 + ($0.04 x P)

Simplifying the equation, we get:

$10.84 = $0.04 x P

Dividing both sides by $0.04, we get:

P = $10.84 / $0.04

Divide the numbers

P = 271

Therefore, Jim ordered 271 prints last month.

Learn more about equation here

brainly.com/question/14603452

#SPJ4

a farmer has 600 yards of fencing to enclose a rectangular garden. express the area a of the rectangle as a function of the width x of the rectangle. what is the domain of a?
a. a(x) = -x^2 + 6000x, Domain = {x|0 < x < 6000 }
b. a(x) = -x^2 + 300x, Domain = {x|0 < x < 300 }
c. a(x) = x^2 + 300x, Domain = {x|0 < x < 300 }
d. a(x) = -x^2 + 300x, Domain = {x|0 < x < 6000 }

Answers

The domain ofA is Answer: "d. [tex]a(x) = -x^2 + 300x[/tex], Domain = {x|0 < x < 6000 "}

A farmer has 600 yards of fencing to enclose a rectangular garden. The area A of the rectangle as a function of the width x of the rectangle can be expressed as follows:

[tex]A = x(300 - x) = 300x - x^{2}[/tex]

The domain of A is given by the interval of values of x that can be chosen for the width of the rectangle. The width of a rectangle must be positive, so we have 0 < x. If the farmer has 600 yards of fencing, we can get the maximum value of x as follows:

600 = 2x + 300x = 600/3 = 200

So we have x < 200.

Therefore, the domain of A is {x | 0 < x < 200}.

Rewriting the equation of A, we get:

A(x) = -x² + 300x, Domain = {x | 0 < x < 200}.

In yards, the answer would be {x | 0 < x < 6000}.

You can learn more about domain at

https://brainly.com/question/26098895

#SPJ11

Function P represents the perimeter, in inches, of a square with the side length x inches.P = x + x + x + x ... but it would be more efficient to write it this way: P =4xComplete the table.x 0 1 2 3 4 5 6P(x) 0

Answers

Function P represents the perimeter, in inches, of a square with the side length x inches. So, the perimeter of a square can be represented as P = x + x + x + x, which simplifies to P = 4x. This way is more efficient to write than the former.

Complete the table provided: x 0 1 2 3 4 5 6 P(x) 0 4 8 12 16 20 24. When x = 0, the perimeter of the square is 0.When x = 1, the perimeter of the square is P = 4(1) = 4. When x = 2, the perimeter of the square is P = 4(2) = 8. When x = 3, the perimeter of the square is P = 4(3) = 12. When x = 4, the perimeter of the square is P = 4(4) = 16. When x = 5, the perimeter of the square is P = 4(5) = 20. When x = 6, the perimeter of the square is P = 4(6) = 24.

Function P represents the perimeter, in inches : https://brainly.com/question/19110581

#SPJ11

In 2008 , the population of a district was 39,700 . With a continuous annual growth rate of approximately 3%, what will the population be in 2033 according to the exponential growth function?

Answers

The population will be approximately 84,161 in 2033 according to the exponential growth function.

The given information in the problem is;Population in 2008 = 39,700

Annual growth rate = 3%

We need to find out the population in 2033.

The formula for continuous exponential growth is;P(t) = P₀e^(rt)

where;P₀ is the initial populationr is the annual growth rate (in decimal form)t is the time elapsed (in years)

We are given P₀ = 39,700r = 0.03t = 2033 - 2008 = 25 years

Put these values in the formula of continuous exponential growth;

P(25) = 39,700e^(0.03 x 25)P(25)

= 39,700e^(0.75)P(25)

= 39,700 x 2.1170000493605122P(25)

= 84,161.13

To know more about exponential growth click on below link:

https://brainly.com/question/12490064#

#SPJ11

Which of these planes is NOT in the {100} family for a tetragonal crystal? (A tetragonal unit cell drawn to proportion is included below for reference.)(A) (010)(B) (001)(C) (110)(D) Both B & C(E) All of these planes are in the {100} family.

Answers

The answer is (001). This is because B (001) has h and k as non-zero integers, which does not match the criteria for the {100} family.

The question is asking which of the planes (A), (B), (C), and (D) is not part of the {100} family for a tetragonal crystal.A tetragonal crystal is a three-dimensional structure made up of four faces that intersect at right angles, forming a unit cell. Each face of the unit cell is defined by a Miller index. A Miller index is a set of three integers written in the form {hkl}, which describes the orientation of the face relative to the crystal lattice. In a tetragonal crystal, the {100} family is the set of faces described by {hkl} such that h = k = 0 and l ≠ 0.



Therefore, A (010), C (110), and E (all of these planes are in the {100} family) are all part of the {100} family for a tetragonal crystal, while B (001) is not.  because B (001) has h and k as non-zero integers, which does not match the criteria for the {100} family. In conclusion, the correct answer to the question is B (001).

To know more about tetragonal crystal system, click here:

https://brainly.com/question/29645362

#SPJ11

Please help me I can figure it out

Answers

Answer:

  (2x -4)/(x -1)

Step-by-step explanation:

You want to simplify (2x² -2x -4)/(x² -1).

Factors

Simplifying a rational expression generally means cancelling common factors from the numerator and denominator. To do that, you must factor both the numerator and the denominator.

Numerator

The numerator coefficients have a common factor of 2. Removing that can simplify the problem of factoring the numerator:

  2x² -2x -4 = 2(x² -x -2)

To factor the quadratic, you look for factors of -2 that have a sum of -1. Those would be -2 and +1. The middle term is written as a sum using these values.

  = 2(x² -2x +x -2)

This can now be factored by grouping terms in pairs, and factoring each pair.

  = 2((x² -2x) +(x -2)) = 2(x(x -2) +1(x -2))

  = 2(x +1)(x -2)

Denominator

The denominator is the difference of squares, so is factored according to the pattern for that:

  a² -b² = (a -b)(a +b)

  x² -1 = (x -1)(x +1)

Simplified form

Now you know enough to be able to simplify the expression. The common factors (x+1) cancel.

  [tex]\dfrac{2x^2-2x-4}{x^2-1}=\dfrac{2(x+1)(x-2)}{(x+1)(x-1)}\\\\=\dfrac{2(x-2)}{x-1}=\boxed{\dfrac{2x-4}{x-1}}[/tex]

__

Additional comment

Even though the factor (x+1) has disappeared from the expression, the simplified form still carries the restriction that x ≠ -1. The graph of the original expression will have a "hole" at (-1, 3), where it is undefined. Otherwise, the graph looks like a graph of (2x-4)/(x-1).

You are dealt five cards from a standard deck of 52 playing cards (A full house consists of three of one kind and two of another. For example, A A A 5-5 and K-K-K 10-10 are full houses) (a) in how many ways can you get a full house? ______ Ways (b) in how many ways can you get a five card combination containing two jacks and three aces ___ ways

Answers

 The 32 ways to get a five-card combination containing two jacks and three aces.

(a) A full house consists of three of one kind and two of another kind. Therefore, there are 13 different choices for the rank of the triplet and 4 cards of the same rank. Once the triplet has been chosen, there are 12 choices for the rank of the pair and 4 cards of the same rank. Therefore, the number of ways to get a full house is as follows:$${13}{\times}{4}{\times}{12}{\times}{4}={7488}$$Therefore, there are 7488 ways to get a full house.(b) In this case, the two jacks and three aces must be chosen out of the 4 jacks and 4 aces in the deck. Therefore, the number of ways to get a five-card combination with two jacks and three aces is as follows:$$\frac{{4\choose2}{4\choose3}{44\choose0}}{5!}={32}$$Therefore, there are 32 ways to get a five-card combination containing two jacks and three aces.

Learn more about Combination

brainly.com/question/27014146

#SPJ11

Other Questions
How did spanish iflunce by destroying there alliance with the chikasaw 6.3 Sunita carries out an experiment to investigate diffusion. She uses waterand a food dye to find out how the volume of water used affects the timethe dye takes to spread evenly through the water.Her prediction is that the more water she uses, the longer the time needed for the dye to spread thoat it. d. How will she ensure her results are reliable For the CPT code assignment 44970, which of the following coding references can be accessed from the coding summary screen? Select all that apply. Coder's Desk Reference for Procedures CPT CPT Assistant DRG Anesthesia Crosswalk HCPCS a block of mass 0.243 kg is placed on top of a light, vertical spring of force constant 4 975 n/m and pushed downward so that the spring is compressed by 0.092 m. after the block is released from rest, it travels upward and then leaves the spring. to what maximum height above the point of release does it rise? (round your answer to two decimal places.) Focus Charting is just one type of documentation that a nurse can use when reporting on psychiatric-mental health patients. Which of the following statements are correct about Focus Charting? Select all that apply.a. Focus Charting uses the DAR format, which stands for Data, Action, Response.The focus can be on nursing diagnosis, current concerns, or significant changes/events.b. Data are any information that supports the focus and are based only on observations.c. Action includes all nursing actions that address the focus and evaluate the care plan.d.Response is the evaluation of the patient responses to the plan of care. DataSource Inc., a data analysis company, uses its performance management system in a more traditional manner with the focus being on using the performance ratings that it generates as the basis for making decisions about merit pay increases, promotions, as well as terminations. This is an example of the ________blank purpose of performance management.Multiple Choicestrategicdevelopmentalsocialacceptabilityadministrative Snowcap Ice Cream produces over a dozen delicious ice cream flavors and a number of individually packaged frozen treats. Which of these strategies would represent vertical integration for Snowcap?Multiple Choicea. purchasing Mountain Dairy milk farmb. acquiring a beverage bottling plantc. opening a personal tax accounting divisiond. setting their prices lower than the competitors prices lifting the weight without a pulley requires a force of 400 newtons over a distance of 4 meters. how do these values change when the pulley is applied? (1 point) responsesA. the force increases, while the distance decreases. B. both the force and the distance decrease. C. the force decreases, while the distance increases. D. both the force and the distance increase. the iso 27005 standard for information security risk management includes five stages including all but which of the following? question 8 options: a. risk assessment b. risk treatment c. risk communication d. risk determination Xochitl spots an airplane on radar that is currently approaching in a straight line, and that will fly directly overhead. The plane maintains a constant altitude of 7425 feet. Xochitl initially measures an angle of elevation of 19 degrees to the plane at point A.At some later time, she measures an angle of elevation of 37 degrees to the plane at point B. Find the distance the plane traveled from point A to point B. Round your answer to the nearest foot if necessary. the post-closing trial balance does not include any revenues, expenses, or dividends, because these accounts are permanent accounts. group startstrue or false true, unselected false, unselected Major League Baseball salaries averaged 1.5 million with a standard deviation of 0.8 million in 1994. Suppose a sample of 100 major league players were taken. Find the approximate probability that the average salary of a hundred players exceeded 1 million thomas thinks it is important to be as efficient as possible and hates to be late to appointments. thomas most likely belongs to a culture. question 13 options: monochronic polychronic low-context high-context collectivistic A. Monochronic cultureB. Polychronic cultureC. Low-Context cultureD. High-Context cultureE. Collectivistic ulture lighting, inc. uses direct labor hours as a basis for allocating overhead. next year's estimated total overhead is $180000 and direct labor hours are predicted to be $30000 hours. the average labor cost is $10 per. what is the predetermined overhead rate there are 24total customers seated at 4 tables in a restaurant each table is the same size and has the same number of customers tell whether each statement is truth or false Students of SOHO Swim Academy are to have extra swim lessons if they cannot swim. The table belowgives Information on students in 4th, 5th & 6th grade.Complete the table 1) how many students need swim lessons?2) How many students are in the 5th grade?3)How many of the 4th graders canswim?4) How many student in 5th & 6thgrade cannot swim?5) How many students attend SOHOSwim Academy?6) How many students attend SOHo swim Academy? 7) What percentage of 5h grade student can swim?8) what percentage of students cannot swim? The law of demand implies, holding everything else constant, that:a. as the price of bagels increases, the quantity of bagels demanded will decrease.b. as the price of bagels increases, the demand for bagels will decrease.c. as the price of bagels increases, the quantity of bagels demanded will increase.d. as the price of bagels increases, the demand for bagels will increase. FILL IN THE BLANK to use a field list to add a field to a report, click the ___ button on the format tab to display a field list. Assume that Maria Gonzalez was 36 years old when she filed her 2022 tax return. She has 2 kids ages 5 & 12 years old who she claimed as dependents on her tax return. Also, her W2 wages were $60,000, $5,000 was received from unemployment income and her federal withholdings were $5,200. She also donated $10,500 to her needy friends and gave $12,000 to her church as tithes. Also, her sales tax was $569. Assume that her tax from the IRS tax table was $5.182. Based on the following facts above, What is Marias?Filing Status ________________Total Income $___________________Adjusted Gross Income (AGI) $______________________Standard Deduction $___________________________Total Itemized Deduction $ ________________________Taxable Income $_______________________________Child Tax Credit (CTC) $ _______________________Tax Refund $____________________________Tax owed, if any $ _________________________ assume a single-issue pipeline. show how the loop would look both unscheduled by the compiler and after compiler scheduling for both floating-point operation and branch delays, including any stalls or idle clock cycles. what is the execution time (in cycles) per element of the result vector, y, unscheduled and scheduled? how much faster must the clock be for proces- sor hardware alone to match the performance improvement achieved by the scheduling compiler? (neglect any possible effects of increased clock speed on memory system performance.)